LSAT and Law School Admissions Forum

Get expert LSAT preparation and law school admissions advice from PowerScore Test Preparation.

User avatar
 Dave Killoran
PowerScore Staff
  • PowerScore Staff
  • Posts: 5862
  • Joined: Mar 25, 2011
|
#87690
Complete Question Explanation
(The complete setup for this game can be found here: lsat/viewtopic.php?t=1469)

The correct answer choice is (B)

This is the third Suspension question of the game, a very high number for a single Games section.

The question asks you to substitute in a new rule for the second rule, while having no impact on determining which courses the student can take. This is a highly restricted request, and there cannot be too many options that will fulfill this request!

To find the correct answer, we must identify the answer choice that rewords the second rule. Thus, there is value in reviewing the language and meaning of the second rule. In the second rule, M is the sufficient condition, and the presence of M indicates that neither P nor T can be taken. The contrapositive asserts that if either P or T is taken, then M cannot be taken. Let’s analyze each answer choice and see which one has the same meaning.

Answer choice (A): This answer is too limited. Whereas the second rule eliminated two variables from contention (but then left several others in contention), this answer just specifies that two specific courses would have to be taken. This is different from the second rule, and so answer choice (A) is incorrect.

Answer choice (B): The pool of variables available to go with M is initially all the other variables: H, L, P, S, T, and W. The first rule eliminated H from consideration, leaving a group of five variables: L, P, S, T, and W. The wording of the second rule eliminated two of these five, P and T. The wording of answer choice (B) simply restates that idea by indicating that the three variables in the group of five besides P and T are eligible to be taken with M. Thus, this answer contains a rule that, although stated differently than the second rule, is still identical in function to the second rule. This answer is therefore correct.

Answer choice (C): At first glance, this answer is unlikely to be correct because it doesn’t address M or T, two elements that appear in the second rule, and are thus affected by it. Although a contrapositive of the second rule indicates that P can’t be taken with M, the variables listed here do not form the group of variables besides M that can be taken with P under the second rule. That alone eliminates this answer, as well as the fact that it does not affect M and T in the same way as the second rule.

Answer choice (D): Similar to answer choice (C), at first glance, this answer is unlikely to be correct because it doesn’t address M or P, two elements that appear in the second rule. Using the same reasoning as in (C), the answer can be eliminated because the second rule allows for a larger group of variables to accompany T than is listed here. The criticism of the effect on M and P also holds.

Answer choice (E): This is a classic trap answer, and one that shows that LSAC knows how to test conditional reasoning. Although a contrapositive of the second rule would be a correct answer, this answer does not supply a contrapositive of the second rule (the “both...and” construction at the start of the sentence should instead be “or”).

Thus, answer choice (B) is correct.
 wwarui
  • Posts: 32
  • Joined: Nov 13, 2011
|
#3458
Thanks. I never deleted my messages after reviewing them. Sorry.
Last edited by wwarui on Tue Dec 17, 2013 10:06 am, edited 1 time in total.
 Adam Tyson
PowerScore Staff
  • PowerScore Staff
  • Posts: 5191
  • Joined: Apr 14, 2011
|
#3461
Hi Wangeci. Question 23 is one of those delightful Rule Substitution question, which asks you to pick the answer that is a rule with the same ultimate effect on the game as the rule they are telling you to get rid of. Let's start by looking at the rule that we are dumping - that if M is selected, then neither P nor T can be selected. What else do we know about what happens when M is selected? We know, based on the contrapositive of the first rule, that H also cannot be selected. So what else is left to go with M in our original scenario? Only three courses are possible with M - L, S, and W.

You're right that S and W can't go together, so there's another inference to be made there: when M is selected, L must also be selected. The third course selection in that case can only be either S or W. So, when M is picked, the solution is either M-L-S or M-L-W. Answer choice B isn't suggesting that S and W can be selected together; it is saying that those are the only three courses from which we can choose two whenever M is selected.

Hope that was clear and helpful. Good luck!

Adam
 wwarui
  • Posts: 32
  • Joined: Nov 13, 2011
|
#3489
Hello Adam,

Oh, thank you so much. I couldn't see the inference.
I'm very grateful.
Wangeci
 Patrick.a.anderson
  • Posts: 4
  • Joined: Jan 28, 2013
|
#22109
Hi, I was confused on what inferences to make for this question, so I had to do each of 6 questions locally. What inferences do you think are key to solving the game? Question #22 gave me particular trouble. To find out this answer, would you make multiple scenarios and just see which solution fits? Also, in question #23, would you just test out each solution if time permits?
User avatar
 Dave Killoran
PowerScore Staff
  • PowerScore Staff
  • Posts: 5862
  • Joined: Mar 25, 2011
|
#22110
Hi Patrick,

Thanks for the question. This is a classic Grouping game, and one that really tests your conditional reasoning abilities.

The first step is to look at each of the rules as being two rules in one. Because of the way each rule is worded, it produces two diagrams. For example, the first rule is really two separate statements:

..... ..... ..... ..... H :dblline: S
..... ..... ..... and
..... ..... ..... ..... H :dblline: M

The problem with all these rules is that they are all negative, and so you can't link them together to make a series of power inferences. So, the good news is that you weren't as confused as you might have thought. There aren't any links to be made, so diagramming the rules is the most you can do as far as inferences.

However, a lack of inferences always makes me suspicious (I don't trust the test makers anyway, but something like this puts me on guard even more). Since diagramming the rules takes no time, and there aren't any inferences to be made, you should take another step during the setup, and that is to look at what happens when each course is taken. That appears as follows:

..... ..... ..... ..... H :arrow: S, M

..... ..... ..... ..... M :arrow: P, T, H

..... ..... ..... ..... P :arrow: M, W

..... ..... ..... ..... S :arrow: H, W

..... ..... ..... ..... T :arrow: M

..... ..... ..... ..... W :arrow: P, S


This process helps clarify all of the relationships, and really crystallizes what can occur. For example, it's easy now to see that selecting M causes the most issues, whereas T is the course that causes the least number of issues. Then, at that point, you are ready to attack the questions.

I'll answer question #23 in just a moment, with a second post.

Please let me know if that helps with the main setup. Thanks!
User avatar
 Dave Killoran
PowerScore Staff
  • PowerScore Staff
  • Posts: 5862
  • Joined: Mar 25, 2011
|
#22111
Ok, the first step in question #23 is to realize what they are asking you to do. This question first suspends the second rule and then asks you to substitute in a new rule for the second rule, while having no impact on determining which courses the student can take (and thus we have a Rule Equivalence question). This is a highly restricted request, and there cannot be too many options that will fulfill this request!

To find the correct answer, we must identify the answer choice that rewords the second rule. Thus, there is value in reviewing the language and meaning of the second rule. In the second rule, M is the sufficient condition, and the presence of M indicates that neither P nor T can be taken. The contrapositive asserts that if either P or T is taken, then M cannot be taken. Let’s analyze each answer choice and see which one has the same meaning.

Answer choice (A): This answer is too limited. Whereas the second rule eliminated two variables from contention (but then left several others in contention), this answer just specifies that two specific courses would have to be taken. This is different from the second rule, and so answer choice (A) is incorrect.

Answer choice (B): The pool of variables available to go with M is initially all the other variables: H, L, P, S, T, and W. The first rule eliminated H from consideration, leaving a group of five variables: L, P, S, T, and W. The wording of the second rule eliminated two of these five, P and T. The wording of answer choice (B) simply restates that idea by indicating that the three variables in the group of five besides P and T are eligible to be taken with M. Thus, this answer contains a rule that, although stated differently from the second rule, is still identical in function to the second rule. This answer is therefore correct.

Answer choice (C): At first glance, this answer is unlikely to be correct because it doesn’t address M or T, two elements that appear in the second rule, and are thus affected by it. Although a contrapositive of the second rule indicates that P can’t be taken with M, the variables listed here do not form the group of variables besides M that can be taken with P under the second rule. That alone eliminates this answer, as well as the fact that it does not affect M and T in the same way as the second rule.

Answer choice (D): Similar to answer choice (C), at first glance, this answer is unlikely to be correct because it doesn’t address M or P, two elements that appear in the second rule. Using the same reasoning as in (C), the answer can be eliminated because the second rule allows for a larger group of variables to accompany T than is listed here. The criticism of the effect on M and P also holds.

Answer choice (E): This is a classic trap answer, and one that shows that LSAC really knows how to test conditional reasoning. Although a contrapositive of the second rule would be a correct answer, this answer does not supply a contrapositive of the second rule (the “both...and” construction at the start of the sentence should instead be “or”).


So, the first note is that this is a very high level question. You can take the brute force method and work through each one, but, you can see that what we tried to do was analyze the nature of what was needed and then find an equivalent, without having to go through the process of drawing out each answer and seeing the concrete effects.

Please let me know if that helps. Thanks!
 jgabalski
  • Posts: 16
  • Joined: Feb 16, 2017
|
#34227
I chose B on this question because of the technical definition of either/or on the LSAT - with the possibility of both. Is that a reason why Answer A is wrong, because it allows for the Possibility of M-S-W, which would go against a rule? Other than that I had trouble picking because A and B because they both felt like logical consequences of the inferences that the original second rule produced. Thank you for the help.
 Charlie Melman
PowerScore Staff
  • PowerScore Staff
  • Posts: 85
  • Joined: Feb 10, 2017
|
#34247
Hi J,

This question gives us the difficult task of picking a new rule that would produce the same set of possibilities that we had before striking the rule the question tells us to strike. In other words, we want to replicate the original rule with a new rule, and they're giving us five possibilities.

Answer choice (A) is incorrect because it says nothing about M not being able to be with P, which was one of the conditions that we originally had. All we know is that we'll have an S or W. What about P?

Answer choice (B) is correct because it excludes all the things we need to exclude. By saying that only L, S, and W are eligible to be with M, we exclude P, T, and H. And then the new world is just the same as the old world.

Hope this helps!

Get the most out of your LSAT Prep Plus subscription.

Analyze and track your performance with our Testing and Analytics Package.